0% found this document useful (0 votes)
33 views4 pages

Solution 2

Uploaded by

Hari Chandan
Copyright
© © All Rights Reserved
We take content rights seriously. If you suspect this is your content, claim it here.
Available Formats
Download as PDF, TXT or read online on Scribd
0% found this document useful (0 votes)
33 views4 pages

Solution 2

Uploaded by

Hari Chandan
Copyright
© © All Rights Reserved
We take content rights seriously. If you suspect this is your content, claim it here.
Available Formats
Download as PDF, TXT or read online on Scribd
You are on page 1/ 4

Department of Mathematics

Indian Institute of Technology Guwahati


MA 101: Mathematics I
Solutions of Tutorial Sheet-2
July-December 2019

1. Let (xn ) be a convergent sequence in R with limit ` ∈ R and let α ∈ R.

(a) If xn ≥ α for all n ∈ N, then show that ` ≥ α.


(b) If ` > α, then show that there exists n0 ∈ N such that xn > α for all n ≥ n0 .
(c) If (xn ) and (yn ) are convergent sequences and xn ≥ yn for all n ∈ N, then
lim xn ≥ lim yn .
n→∞ n→∞

(Note that ` can be equal to α in (a) even if xn > α for all n.)

Solution. (a) If possible, let ` < α. Then α − ` > 0 and since xn → `, there exists
n0 ∈ N such that |xn − `| < α − ` for all n ≥ n0 . This implies that xn < ` + α − ` = α
for all n ≥ n0 , which is a contradiction. Hence ` ≥ α.
(b) Since ` − α > 0 and since xn → `, there exists n0 ∈ N such that |xn − `| < ` − α
for all n ≥ n0 . This implies that xn > ` − (` − α) = α for all n ≥ n0 .
(c) We have xn − yn ≥ 0 for all n. The result follows readily from part (a).

1 1
(Note that although n
> 0 for all n ∈ N, lim = 0 and thus ` can be equal
n→∞ n
to α in (a) even if xn > α for all n.)

2. Let (xn ) be a convergent sequence of positive real numbers such that lim xn < 1.
n→∞
Show that lim xnn = 0.
n→∞

Solution. If ` = lim xn , then ε = 12 (1 − `) > 0 and so there exists n0 ∈ N such


n→∞
that |xn − `| < 12 (1 − `) for all n ≥ n0 . Hence 0 < xn < 21 (1 + `) for all n ≥ n0
⇒ 0 < xnn < ( 1+`
2
)n for all n ≥ n0 . Since 12 (1 + `) < 1, lim ( 1+` )n = 0. Therefore by
n→∞ 2
Sandwich theorem, lim xnn = 0.
n→∞

3. If |α| < 1, then the sequence (αn ) converges to 0.

Solution. If α = 0, then αn = 0 for all n ∈ N and so (αn ) converges to 0. Now we


1 1
assume that α 6= 0. Since |α| < 1, |α| > 1 and so |α| = 1 + h for some h > 0. For all
n ∈ N, we have

n(n − 1) 2
(1 + h)n = 1 + nh + h + · · · + hn > nh.
2!
1 1
This implies |α|n = (1+h) n < nh for all n ∈ N. Given ε > 0, we choose n0 ∈ N
1
satisfying n0 > hε . Then |αn − 0| = |α|n < n10 h < ε for all n ≥ n0 and hence (αn )
converges to 0.

log ε
Alternative proof: Given ε > 0, we choose n0 ∈ N satisfying n0 > log |α|
. Then for
n n n0 n
all n ≥ n0 , we have |α − 0| = |α| ≤ |α| < ε and hence (α ) converges to 0.
1
4. Show that the sequence ((2n + 3n ) n ) converges to 3.
1 1
Solution. We have 3n < 2n + 3n < 2 · 3n for all n ∈ N. Hence, 3 < (2n + 3n ) n < 2 n · 3
1
for all n ∈ N. Since 2 n → 1 (done in the class), hence by Sandwich theorem, the
given sequence converges to 3.

5. Let (an ) be a sequence of real numbers such that each of the subsequences (a2n ),
(a2n−1 ) and (a3n ) converges. Show that (an ) is convergent.

Solution. Let a2n → a, a2n−1 → b and a3n → c. Clearly, (a6n ) is a subsequence of


(a2n ) and (a3n ). Hence, a6n → a and a6n → c. This implies a = c.
Again, (a3(2n−1) ) is a subsequence of (a2n−1 ) and (a3n ). Hence, a3(2n−1) → b and
a3(2n−1) → c. This implies b = c, and hence a = b = c. Since (a2n ) and (a2n−1 )
converge to the same limit, it follows that (an ) is convergent.

6. If (an ) is a bounded sequence and (bn ) is another sequence which converges to 0,


show that the product (an bn ) converges to 0.

Solution. Since (an ) is bounded, so there is a positive number M such that |an | ≤ M
for all n ∈ N. Let ε > 0. Since (bn ) converges to 0, so for given ε/M > 0, there
exists n0 ∈ N such that |bn | < ε/M for all n ≥ n0 . Now, for n ≥ n0 , we have
ε
|an bn | = |an ||bn | < M · ⇒ |an bn | < ε for all n ≥ n0 .
M
This proves that an bn → 0.

Remark 1. Note that if (an ) is not bounded then the result need not be true. For
example, take an = n2 and bn = n1 .

1P n
7. Let (an ) be a sequence of real numbers. Define the sequence (sn ) by sn = ai .
n i=1
(a) If (an ) is bounded, then show that (sn ) is also bounded.

Solution. We have |an | ≤ M for all n ∈ N. Then we obtain |sn | ≤ M for all
n ∈ N.

(b) If (an ) is monotone, then show that (sn ) is also monotone.

Solution. We have
n+1 n n
1 X 1X an+1 1 X
sn+1 − sn = ai − ai = − ai
n + 1 i=1 n i=1 n + 1 n(n + 1) i=1

Now, if (an ) is increasing, then an+1 ≥ ai for all i = 1, 2, . . . , n.


Hence, sn+1 ≥ sn for all n. Similarly, if (an ) is decreasing, then an+1 ≤ ai for
all i = 1, 2, . . . , n. Hence, sn+1 ≤ sn for all n.

(c) If (an ) converges to `, then show that the sequence (sn ) also converges to `.
Solution. Given that an → `. We have
n n
1X 1X
|sn − `| = | ai − `| ≤ |ai − `|.
n i=1 n i=1

Let ε > 0. Since an → `, so there exists a positive integer n0 such that


|ai − `| < ε/2 for all i ≥ n0 . Hence, for n ≥ n0 we have
n n0 −1 n
1X 1 X 1X α
|sn − `| ≤ |ai − `| = |ai − `| + |ai − `| < + ε/2,
n i=1 n i=1 n i=n n
0

Pn0 −1 α
where α = i=1 |ai − `|. Since n → 0, there exists a positive integer n1
such that αn < ε/2 for all n ≥ n1 . Hence, |sn − `| < ε for all n ≥ n2 , where
n2 = max{n0 , n1 }.

8. Show that the sequence (xn ) defined by xn = 1 + 12 + 13 + · · · + n1 diverges to infinity.

Solution. Clearly, (xn ) is an increasing sequence. Also, for n ∈ N, we have


     
1 1 1 1 1 1 1 1 1
x2n = 1 + + + + + + + + ··· + + ··· + n
2 3 4 5 6 7 8 2n−1 + 1 2
n−1
1 2 4 2
≥ 1 + + + + ··· + n
2 4 8 2
n
=1+ .
2
This proves that (xn ) is not bounded above. Hence, (xn ) diverges to infinity.

9. Let the sequence (an ) be defined by


1/2
3 + a2n

a1 = 1, an+1 = , n ≥ 1.
2

Show that (an ) converges to 3.

Solution. Using the principle of mathematical induction, we find that an ≤ 3 for
2
all n ≥ 1. Also, an > 1 for all n. We now find that a2n+1 − a2n = 32 − a2n ≥ 0, and
hence an+1 ≥ an for all n. This proves that
√ the sequence is convergent. Let xn → `.
2
Then, ` = 3. Since ` is positive, so ` = 3.
 
1 2
10. Let a1 > 0 and for n ≥ 1, an+1 = an + . Show that the sequence {an } is
2 an
convergent and find the limit.
√ √ √
Solution. Since a1 > 0, we can write an+1 = 12 (an + a2n ) = 12 ( an − √a2n )2 + 2.

√ implies an+1 ≥ 2 for all n ∈ N. √
This Thus, (an ) is bounded below. Note that
2 need not be a lower bound. If a1 < 2, then a1 will be a lower bound. Now,
2
2an+1 − an = a2n . This implies 2an+1 − 2an = a2n − an = 2−a an
n
≤ 0 for all n ≥ 2.
Thus,
a2 ≥ a3 ≥ · · · ≥ an ≥ · · ·

Hence (an ) is convergent. If an → `, then `2 = 2. Hence, ` = 2.
1
11. For a ∈ R, let x1 = a and xn+1 = (x2n + 3) for all n ≥ 2. Examine the convergence
4
of the sequence {xn } for different values of a. Also, find lim xn whenever it exists.
n→∞

Solution. If {xn } converges, then ` = lim xn satisfies `2 − 4` + 3 = 0. Hence ` = 1


or ` = 3.
We have xn+1 − xn = 41 (x2n − x2n−1 ) for all n > 1. Also x2 − x1 = 14 (a − 1)(a − 3).
Case 1: If a > 3 then x2 > x1 and we get xn+1 > xn for all n. If {xn } converges,
then ` = lim xn = sup{xn : n ∈ N} ≥ x1 = a > 3, which is not possible. Hence, if
a > 3 then {xn } can’t converge.
Case 2: If a = 3, then xn = 3 for all n ∈ N, and hence {xn } converges to 3.
Case 3: If 1 < a < 3, then x2 < x1 and we get xn+1 < xn for all n ∈ N. Also in this
case xn > 1 for all n ∈ N. (Because xn+1 − 1 = 14 (x2n − 1) for all n ∈ N and x1 > 1.)
Hence {xn } converges to 1. Note that xn 6→ 3 as lim xn = inf{xn : n ∈ N} ≤ x1 =
a < 3.
Case 4: If 0 ≤ a ≤ 1, then x2 ≥ x1 and we get xn+1 ≥ xn for all n ∈ N. Also in
this case xn ≤ 1 for all n ∈ N. Hence {xn } converges to 1.
Case 5: The case for a < 0 is treated by considering −a in place of a, because x2 is
same irrespective of whether we choose x1 = a or x1 = −a. Hence we can say that
for −1 ≤ a ≤ 0, xn → 1, for −3 < a < −1, xn → 1, for a = −3, xn → 3 and for
a < −3, {xn } does not converge.

12. Let x1 = 6 and xn+1 = 5 − x6n for all n ∈ N. Examine whether the sequence (xn ) is
convergent. Also, find lim xn if (xn ) is convergent.
n→∞

Solution. We have x1 > 3 and if we assume that xk > 3 for some k ∈ N, then
xk+1 > 5 − 2 = 3. Hence by the principle of mathematical induction, xn > 3 for
all n ∈ N. Therefore (xn ) is bounded below. Again, x2 = 4 < x1 and if we assume
that xk+1 < xk for some k ∈ N, then xk+2 − xk+1 = 6( x1k − xk+11
) < 0 ⇒ xk+2 <
xk+1 . Hence by the principle of mathematical induction, xn+1 < xn for all n ∈ N.
Therefore (xn ) is decreasing. Consequently (xn ) is convergent. Let ` = lim xn .
n→∞
6 6
Then lim xn+1 = 5 − lim xn
⇒ ` = 5− `
(since xn > 3 for all n ∈ N, ` 6= 0)
n→∞ n→∞
⇒ (` − 2)(` − 3) = 0 ⇒ ` = 2 or ` = 3. But xn > 3 for all n ∈ N, so ` ≥ 3.
Therefore ` = 3.

You might also like